Processing math: 2%
Skip to main content

Posts

Showing posts from September, 2021

Some random problems

  I know, I know. Different font indeed. I have deleted a few of my MSE answers. I felt they weren't that good in quality. And a few questions are from my prev aops account which I have deactivated now. I also have posted 10 IOQM types of problems. These can be used while preparing for IOQM. Problem: Prove that \dfrac{ab}{c^3}+\dfrac{bc}{a^3}+\dfrac{ca}{b^3}> \dfrac{1}{a}+\dfrac{1}{b}+\dfrac{1}{c}, where a,b,c  are different positive real numbers.  Proof: Note that by AM-GM \frac{ab}{c^3}+\frac{bc}{a^3}\ge \frac{2b}{ac} and we also have \frac {b}{ac}+\frac{c}{ab}\ge \frac{2}{a}. Hence, \sum_{cyc}\frac{ab}{c^3}\ge\sum_{cyc}\frac{b}{ac}\ge\sum_{cyc}\frac{1}{a} where everything we got is by applying AM-GM on 2 terms and then dividing by 2. USA TST 2007: Triangle ABC which is inscribed in circle \omega. The tangent lines to \omega at B and C meet at T. Point S lies on ray BC such that AS is perpendicular to AT. Points B_1 and $C_1...

Problems done in August

  Welcome back! So today I will be sharing a few problems which I did last week and some ISLs. Easy ones I guess. Happy September 2021!  Problem[APMO 2018 P1]: Let ABC be a triangle with orthocenter H and let M and N denote the midpoints of {AB} and {AC}. Assume H lies inside quadrilateral BMNC, and the circumcircles of \triangle BMH and \triangle CNH are tangent. The line through H parallel to {BC} intersects (BMH) and (CNH) again at K, L respectively. Let F = {MK} \cap {NL}, and let J denote the incenter of \triangle MHN. Prove that FJ = FA. Proof:  By angle chase, we get \angle FKL=\angle FLK.    Hence KL||MN\implies FM=FN.   And we get \angle MFN=2A\implies F is circumcentre if (AMN)\implies FA=FM=FN.   And we get \angle MHN=180-2A    Hence MFHN is cyclic.    By fact 5, ME=FJ=FN\implies FJ=FA. Problem[Shortlist 2007 G3]: Let ABCD be a trapezoid whose diagonals meet at P....